LSAT and Law School Admissions Forum

Get expert LSAT preparation and law school admissions advice from PowerScore Test Preparation.

User avatar
 Dave Killoran
PowerScore Staff
  • PowerScore Staff
  • Posts: 5848
  • Joined: Mar 25, 2011
|
#85362
Complete Question Explanation
(The complete setup for this game can be found here: lsat/viewtopic.php?f=315&t=9170)

The correct answer choice is (E)

If J views the site on day 1, then as discussed during the setup the following diagram results:

G1-Q2-d1.png

Accordingly, answer choice (E) is correct. Note that M was our last variable placed in our discussion, and not surprisingly, it is the correct answer here.
 studyhelp20
  • Posts: 28
  • Joined: Dec 09, 2020
|
#82390
Hello Power Score Support Staff,

Could I please receive some help in explaining the correct answer to this problem? Also, could I have explanations for all of the incorrect answers. Especially why answer C is incorrect? Thanks for the help

Sincerely,
Brennan
User avatar
 KelseyWoods
PowerScore Staff
  • PowerScore Staff
  • Posts: 1079
  • Joined: Jun 26, 2013
|
#82416
Hi Brennan!

As I said in over on the board for the setup to this game, there is only one possible order that works if J is first. See if you can figure it out!

If J is first, H is second. Now what? What other rules come into play here? Think about the HG rule. Where would G have to go if H is second? What happens with the F rule? What options are left for L? Now think about what you know about K :longline: this one is tricky but remember to use your contrapositive for the K4 :arrow: L5 rule. Now there's only one slot left for M. See if you can figure it out for yourself first and then let us know if/where you get stuck!

Hope this helps!

Best,
Kelsey

Get the most out of your LSAT Prep Plus subscription.

Analyze and track your performance with our Testing and Analytics Package.